1
$\begingroup$

I know that similar questions have been answered several times, and based on the answers, I attempted a solution to my problem. But I simply do not get the right results. The problem is as follows. I wish to solve the following optimization:

$max(ax_1)$ subject to $x_1 = |x_2-x_3|$

where $a$ can be both positive and negative.

I tried to set up the a "Big-M" method based Binary LP (The bounds are well known and reasonably small.) by using the following constraints:

$x_1+(x_2-x_3)\leq M*p$

$x_1-(x_2-x_3)\leq M*(1-p)$

$x_1+(x_2-x_3)\geq -M*p$

$x_1-(x_2-x_3)\geq -M*(1-p)$

where p is binary.

The bounds are $0 \leq x_1 \leq U$, where $U << M$; $-\infty \leq x_2 \leq \infty $; $-\infty \leq x_3 \leq \infty$

But this did not work. Is my above approach correct? Am I doing something wrong because of non-convexity and because of lack of closed-form solutions (as explained in Linear programming with absolute values)?

If yes, are there any alternatives? I am stuck!

This is the theoretical part.

In practice, I am having an issue with the following problem. $max(x_1(1)+x_1(2)+x_1(3))$

such that

$x_1(1) = |x_2(2)-x_2(1))|$

$x_1(2) = |x_2(3)-x_2(2))|$

$x_1(3) = |x_2(4)-x_2(3))|$

the bounds are $0\leq x_1(1) \leq 4$, $0 \leq x_1(2) \leq 12$, $0\leq x_1(3) \leq 11$. for $x_2(1,...,4)$, the bounds are [-inf inf].

I expect the fval to be 27 so that I get x_1(1,...,3) = 4, 12, 11. but I get fval to be 4 and x_1(1,...,3) = 4, 0, 0.

I suppose I am making a big blooper somewhere, but I cant figure out where...!

$\endgroup$

0

Your Answer

By clicking “Post Your Answer”, you agree to our terms of service and acknowledge you have read our privacy policy.

Start asking to get answers

Find the answer to your question by asking.

Ask question

Explore related questions

See similar questions with these tags.